You are on page 1of 112
PRACTICAL NOTES Pharmacology 3 Suez canal university Faculty of pharmacy Department of pharmacology and toxicology 2021/2022 Contents: Intended learning outcomes Dyspepsia .......... Inflammatory bowel diseases 30, Constipation... Diarrhea... Oral hypoglycemic agents 1 Diabetes cases........ Hyper and hypothyroidism. ... Corticosteroids. ....... puseiarseutes zee Lig ‘ (il OD Adrenocortical disorders... . Intended learning outcomes: By the end of the course, the student should be able to: 1. Explain causes and pathophysiology and symptoms of some diseases affecting adrenal gland, thyroid gland, renal system and GIT. 2. Illustrate the pharmacotherapy- ted problems such as adverse effects, interactions, disease limitations and contrandictions 3. Identify problems met with drug use and management of disease of special population. 4. List the underlying principles of proper choice of drugs in treating major diseases affecting GIT and endocrine glands' disorders. © It refers to multiple heterogenous symptoms located in the upper abdomen. © Symptoms included under the umbrella of dyspepsia term include but not limited to © Epigastric pain / burning © Post prandial fullness/early satiety * Belching © Nausea / vomiting © Heart burn / regurgitation Uninvestigated dyspepsia epigastric pain, epigastric burning) Endoscopy, other investigations Epidemiology and risk factors * It is one of the commonest gastrointestinal disorders affecting at least 25% of the population during a year. * ts prevalence varies in different countries, depending upon the prevalence of: Helicobacter pylori (H. pylori) infection Obesity. Drug. Alcohol. Tobacco intake and diet (spices , fat content), Organic Dyspepsia * Most common causes include PUD and GERD. © Can also be caused by gastric erosions, acute or chronic gastritis, duodenal ulcer, mesenteric ischemia , malignancy , medication. © Evidence of an organic disease is observed on upper gastrointestinal endoscopy (and gastric biopsy), or barium imaging. Esophagus: Hemiated porton of stomach Diaphragm’ ‘Stomach- NORMAL STOMACH SLIDING HIATAL HERNIA PARAESOPHA‘ GEAL (ROLLING) HIATAL HERNIA GERD Occur When: the gastroesophageal sphincter is weak, e.g. In the presence of a hiatal hernia, Where sphincter muscle tone is reduced by drugs such as anticholi ;, theophylline and calcium channel blockers. ‘The acid contents of the stomach can leak backwards into the esophagus. The symotoms arising are typically described as heartburn, but many patients use the terms heartburn and dyspepsia interchangeably. Dyspepsia versus GERD To differentiate between Hiatal Hernia dyspepsia and GERD, the Hernia — r Malignancy : Persisting upper abdominal pain, especially when associated with anorexia and unexplained weight loss, may herald an underlying cancer of the stomach or panereas > Gastroparesis can be idiopathic or occur in diabetes mellitus. The risk of gastric or esophageal malignancy in patients with dyspeptic symptoms is estimated to be less than 1%, The risk of gastric cancer is increased among persons with Hp infection, a family history of gastric malignancy, or a history of gastric surgery or immigration from an area endemic for gastric malignancy. The risk of esophageal cancer is increased in men, smokers, persons with high alcohol consumption, and those with a long-standing history of heartburn The symptom pattern associated with gastroparesis (idiopathic, drug-induced, or secondary to metabolic, systemic, or neurologic disorders) is similar to dyspepsia, and the distinction between idiopathic gastroparesis and functional dyspepsia with delayed gastric emptying is not well defined may warrant motility study, Systemic causes Infections, IBD, IBS, and pancreatic and gall bladder disorders are other causes for dyspepsia Systemic Conditions Adrenal insufficiency Diabetes mellitus Heart failure, myocardial ischemia Intra-abdominal malignancy Pregnancy Renal insufficiency Thyroid disease, hyperparathyroi Pancreatic and Biliary Tract Disorders Despite the high prevalence of both dyspepsia and gallstones in adults, epidemiologic studies have confirmed that cholelithiasis is not associated with dyspepsia. Persons with dyspepsia should not be routinely investigated for cholelithiasis, and cholecystectomy in persons with cholelithiasis is not indicated for dyspepsia alone. The clinical presentation of biliary pain is easily distinguishable from that of dyspepsia (see Chapter 65). Pancreatic dis s, but symptoms of acute or chronic pancreatitis or of pancreatic cancer may initially be mistaken for dyspepsia. Pancreatic disorders are usually associated with more severe pain and ofien accompanied by anorexia, rapid weight loss, or jaundice. ase is less prevalent than cholelithias Jaundice occurring in patients with pancreatitis is usually due to hepatocellular injury or to associated biliary tract disease. Common duet obstruction is occasionally caused by pancreatic fibrosis, edema or pseudocyst in patients who have neither hepatocellular injury nor biliary tract disease. Biliary pain from gallstones is usually distinguishable from other causes of epigastric pain by its severity, unpredictability, radiation, and course (lasting houts) it is not usually colicky Despite symptoms of pancreatic and bladder disease are different they may be mistaken by patient with dyspepsia or co-occur. Intolerance to food Historically and in common belief it has been thought that ingestion of some types of food (coffee , alcohol , spices ) or large amount of foods or drugs is responsible for the occurrence of dyspepsia. Recent studies shows that these factors can aggravate dyspeptic symptoms but are not considered the cause themselves These foods appear to affect the sensory and motor function in patients suffering from dyspepsia. Acute ingestion of capsaicin induces dyspeptic symptoms in healthy persons and those with functional dyspepsia, with greater intensity in the latter group Drug induced dyspepsia Medications can cause dyspepsia by number of mechanisms which include: * Alteration in sensory and motor function * Provocation of gastroesophageal reflux * Idiosyneratic mechanisms Medications causing dyspepsia Acarbose ‘Asprin and other NSAIDs (including COX-2 selective agents) Colchicine Digitalis preparations Estrogens Ethano. Gemfitcozi Glucocorticoias Levedepa Nareotes, Niacin Oriistat Potassium chloride Quinaine Sildenati Theophyline Functional dyspepsia Includes 1 or more of the following: © Bothersome postprandial fullness + Early satiation © Epigastric pain + Epigastric burning And no evidence of structural disease (including at upper endoscopy) that is likely to explain the symptoms Although often chronic, symptoms in functional dyspepsia are mostly intermittent. cven during highly symptomatic periods. In persons with functional dyspepsia who present for tertiary care, the most frequent symptoms are postprandial fullness and bloating, followed by epigastric pain, early satiation, nausea, and belching.3,17-20 There is considerable heterogeneity, as demonstrated, for example, by the number of symptoms patients report. Rome IV Diagnostic Criteria for Dyspepsia Must fulfill both the following criteria for the last 3 months with symptom onset >6 months ago: 21 of the following: * Bothersome postprandial fullness. * Bothersome early satiation, © Bothersome epigastric pain © Bothersome epigastric burning No evidence of organic, systemic, metabolic or structural disease (including on upper endoscopy) that is likely to explain the symptoms. Dyspepsia seen at a tertiary care center and persons with uninvestigated dyspepsia in the general population have revealed that between 40% and 75% of dyspeptic persons. Report aggravation of symptoms after ingestion of a meal.23,31,32. Assuming a distinction between meal-related and meal unrelated symptoms might be pathophysiologically and clinically relevant, the Rome III Consensus Committee proposed that functional dyspepsia be used as an umbrella term and 10 that postprandial distress syndrome (PDS)—characterized by mealrelated dyspeptic symptoms, postprandial fullness, and early satiation—be distinguished from epigastric pain syndrome (EPS), characterized by meal-unrelated dyspeptic symptoms, epigastric pain, and epigastric burning. Postprandial Distress Syndrome Must include 1 or both of the following: > Bothersome postprandial fullness, occurring after ordinary sized meals, at least several times per week > Early satiation that prevents finishing a regular meal, at least several times per week. Supportive Criteria 1. Upper abdominal bloating or postprandial nausea or excessive belching can be present 2. Epigastric pain syndrome may coexist The Rome II committee proposed a classification based on predominant symptom of pain or discomfort. Although correlations were found between this classification and the presence or absence of Hp infection, absence or presence of delayed gastric emptying, and response or lack of response to acid-suppressive therapy, the classification has been criticized because of the difficulty in distinguishing pain from discomfort, lack of a widely accepted definition of “predominant,” uncertainty about overlap between the symptom subgroups, lack of an association with putative pathophysiologic mechanisms, and especially lack of stability of the predominant symptom over short time periods. Epigastric Pain Syndrome Must include all the following: * Pain or burning localized to the epigastrium of at least moderate severity, at least once per week. © Pain is intermittent 1 © Not generalized or localized to other abdominal or chest regions. © Not relieved by defecation or passage of flatus. © Not fulfilling criteria for gallbladder or pancreatic disorders Functional dyspeps (ED) Defined according to Rome IV Diagnostic Criteria for Dyspepsia. Use in patients with recurrent upper GI symptoms on average once weekly in the last 3 months with symptom onset >6 months ago and no abnormalitics on diagnostic testing, including upper endoscopy. Do NOT use in patients with alarm symptoms Other GI conditions (such as gastroesophageal reflux disease (GERD), irritable bowel syndrome (IBS), ete) may coexist with functional dyspepsia. Pathophysiology of FD 1- Delayed gastric emptying When the GI tract works normally, the stomach should be 90 percent empty within four hours of eating a meal. Patients suffering from Delayed gastric emptying report symptoms of early satiation, postprandial fullness, nausea and vomiting, 2 2- Impaired gastric accommodation to meal normally the proximal stomach acts as reservoir for ingested food while the distal stomach acts as site for digestion. Structure of the stomach sphincter uodenem Proximal ‘stomach Dsta Failure of vagally mediated relaxation Oy of the proximal stomach in response to increased gastric pressure with food. Antrum: Insufficient accommodation of the proximal stomach activates mechanoreceptors which induces the symptoms The motor functions of the proximal and distal stomach differ remarkably. Whereas the distal stomach regulates gastric empting of solids by grinding and sieving the content until the particles are small enough to pass through the pylorus, the proximal stomach serves mainly as a reservoir during and after ingestion of a meal. Accommodation of the stomach to a meal results from vagally mediated reflex relaxation of the proximal stomach, thereby enabling the stomach to handle large intragastric volumes without a rise in intragastric pressure. Studies using intragastric manometry have shown that ingestion of a meal is associated with a drop in intragastric pressure followed by gradual recovery of the pressure during continued ingestion of nutrients, with increasing meal-induced satiation. Studies using a gastric barostat, scintigraphy, US single photon emission CT (SPECT), or noninvasive surrogate markers (satiation drinking test) have all identified impaired gastric accommodation in roughly 40% of patients with functional dyspepsia. Insufficient accommodation of the proximal stomach during and after ingestion of a meal may be accompanied by increased intragastric pressure and activation of mechanoreceptors in the gasirie wall, thus inducing symptoms. Although a number of studies have found an association between impaired accommodation and early satiation or weight loss, others have failed to find such an asso ‘The mechanisms by which impaired accommodation can cause symptoms is still unclear, Meal ingestion in the absence of proper relaxation of the proximal stomach may be accompanied by activation of tension-sensitive mechanoreceptors in the proximal stomach. On the other hand, insufficient accommodation of the proximal stomach may force the meal into the distal stomach, thereby causing activation of tension-sensitive mechanoreceptors in a distended Antrum. 3. Hypersensitivity to distension of the stomach or duodenum (visceral hypersensitivity) 4. Increased sensitivity off duodenum to lipids or acids. 5. Psychological distress is common in patients with functional dyspepsia but may begin after the gut symptoms manifest, Anxiety is prevalent, but depression can occur and should not be missed. Visceral hypersensitivity, defined as abnormally enhanced perception of visceral stimuli, is considered 1 of the major pathophysiologic mechanisms of all functional Gl disorders. 4 Several studies have established that, as a group, patients with functional dyspepsia are hypersensitive to isobaric gastric distention In healthy persons and patients with functional dyspepsia, duodenal perfusion with nutrient lipids, but not glucose, enhances the perception of gastric distention through a mechanism that requires lipid digestion and the subsequent release of cholecystokinin. Duodenal infusion of hydrochloric acid induces nausea in patients with functional dyspepsia but not in healthy subjects, suggesting duodenal hypersensitivity to acid. Duodenal pH monitoring with a clipped pH electrode has revealed increased postprandial duodenal acid exposure in patients with functional dyspepsia compared with controls, and this difference has been attributed to impaired clearance of acid from the duodenum People with postprandial distress have unique duodenal pathology, namely increased duodenal eosinophils that may degranulate, Duodenal eosinophils have been linked to increased mucosal permeability, submucosal neuronal structural and functional changes, and symptoms.They may reflect an infectious or allergenic trigger. In functional dyspepsia, the duodenal microbiome is also abnormal with increased oral streptococci. H.Pyori Most patients with H. pylori do not develop functional dyspepsia so in many of these cases it is an incidental finding. However in a minority, eradicating the infection cures dyspepsia long term, especially in those with epigastric pain as the main problem Other cause: genetic. Post-infection Functional Dyspepsia: the most common psychiatric comorbidities in patients with functional dyspepsia are anxiety, depressive or somatoform disorders, and a recent or remote history of physical or sexual abuse 15 Although these observations show a close interaction between different psychosocial variables and the presence and severity of symptoms of functional dyspepsia, they do not establish whether psychosocial factors and functional dyspepsia are separate manifestations of a common predisposition or whether psychosocial factors play a causal role in the pathophysiology of dyspeptic symptoms. The relationship is unlikely to be simple. A factor analysis of symptoms of functional dyspepsia and their relationship with pathophysiologyand psychopathology has demonstrated the heterogeneity and complexity of these interactions. Proposed disease model Recently a unifying disease model has been proposed for functional dyspepsia Either an infection, microbiome alteration or a food allergen, such as wheat, induces increased duodenal permeability and duodenal eosinophilia with or without increased mast cells. This activates a mucosal immune response. Local duodeno-gastric reflex responses to low-grade inflammation alter gastroduodenal function, including impaired fundic relaxation in a subset of patients. Circulating cytokines such as tumour necrosis factor alpha may lead to systemic and central nervous system symptoms such as anxiety.These concepts are all supported by experimental evidence and, if correct, the model represents a paradigm shift with profound treatment implications. Alarm Features Alarm features suggesting the need to search for an organic cause of chronic dyspepsia , where use of upper GIT endoscopy is advised include: Age > 60 ycars at onsct; Family history of GI malignancy; Worsening dysphagia: Worsening weight lo: Gl bleeding:/anemia; yen we 16 6. Sudden change of bowel habit; 7. Persistent vomiting. Weight loss is traditionally considered an “alarm” symptom, pointing to potentially serious organic disease Studies in tertiary care patients with functional dyspepsia have also shown a high frequency of unexplained weight loss, and population-based studies in Australia and Europe have shown an association between uninvestigated dyspepsia and unexplained weight loss. Management and diagnosis The initial aim of management is to decide which patients can be treated empirically and which patients should be referred for additional diagnostic evaluation. Exclude organic disorders with appropriate history taking, physical examination, laboratory and endoscopic examination when advised. Ifno organic disorders is detected apply Rome diagnostic criteria Once FD is diagnosed carry treatment following appropriate guidelines. Taking into account the high prevalence of dyspepsia and the large number of persons who present to a physician for their symptoms, the initial aim of management is to decide which patients can be treated empirically and which patients should be referred for additional diagnostic evaluation. Both IBS and FD can overlap and / or exist together: The possible presence of overlapping IBS should also be assessed, and symptoms that improve with bowel movements or are associated with changes in stool frequency or consistency should lead to a presumptive diagnosis of IBS. The nature, frequency, and chronicity of the symptoms, as well as their relationship to meals and the possible influence of specific dietary factors, should be assessed 7 The onset of symptoms—acute with a gastroenteritis-like episode or more gradual—is also of interest. The presence and degree of weight loss, if present, must be determined, as should other alarm symptoms like blood loss and dysphagia, as well as anemia. Distinguishing the EPS from the PDS symptom subgroup according to the Rome III classification may influence the choice of treatment. Assessment of symptoms or signs of a systemic disorder (e.g., diabetes mellitus, cardiac disease, thyroid disorders) and of the patient’s family and personal history will indicate whether the patient is at risk for a particular organic disease that may present as dyspepsia. Laboratory tests Most clinicians will consider routine tests (complete blood count, serum electrolytes, calcium, liver biochemical tests, and thyroid function) after the age of 45 to $5. Other studies like a serum amylase level, antibodies for celiac disease, stool testing for ova and parasites and for Giardia antigen, and a pregnancy test may be considered in selected case The cost-effectiveness of routine laboratory testing, especially in younger patients with uncomplicated dyspepsia, has not been established. The nature, frequency, and chronicity of the symptoms, as well as their relationship to meals and the possible influence of specific dietary factors, should be assessed. The onset of symptoms—acute with a gastroenteritis-like episode or more gradual—is also of interest. The presence and degree of weight loss, if present, must be determined, as should other alarm symptoms like blood loss and dysphagia, as well as anemia, presence of these necessitates investigation (endoscopy and laboratory tests.) Physical findings such as an abdominal mass, organomegaly, ascites, or a posi fecal occult blood test result warrant further evaluation, 18 Burning pain confined to the epigastrium is a cardinal symptom of dyspepsia and not considered heartburn unless it radiates retrosternally. The presence of frequent and typical reflux symptoms should lead to a provisional diagnosis of GERD rather than dyspepsia, and the patient should be treated initially for GERD. The possible presence of overlapping IBS should also be assessed, and symptoms that improve with bowel movements or are associated with changes in stool frequency or consistency should lead to a presumptive diagnosis of IBS. Laboratory testing The cost-effectiveness of routine laboratory testing, especially in younger patients with uncomplicated dyspepsia, has not been established. Nevertheless, most clinicians will consider routine tests (complete blood count, serum electrolytes, calcium, liver biochemical tests, and thyroid function) afier the age of 45 to 55. Other studies like a serum amylase level, antibodies for celiac disease, stool testing for ova and parasites and for Giardia antigen, and a pregnancy test may be considered in selected cases. Diagnosis in those with no risk factors for organic disease. The optimal management strategy for the majority of patients who do not have a risk factor for an organic cause of dyspepsia remains a matter of debate and controversy; several approaches have been proposed. Available options include prompt diagnostic endoscopy followed by targeted medical therapy; noninvasive testing for Hp infection, followed by treatment based on the result (“test and-treat” strategy); and empirical antisecretory drug therapy. In the 2 latter strategies, endoscopy is performed in patients who do not respond to treatment or experience recurrent symptoms after treatment. In theory, empirical therapy with a prokinetic agent could also be considered as an initial option but is generally not recommended because of the lack of widely available prokinetic drugs with established efficacy Patients should receive endoscopy if there is a family history 19 of cancer or population has high prevalence of cancer or patient has anxiety of symptoms. When to carry endoscopy? Patient presenting with dyspeptic symptoms: © [age 60 or older/alacm signs : carry EGD * If on NSAIDs or COX-2 inhibitor : discontinue /switch / add PPI/carry endoscopy to exelude ulcer. * If younger than 60 with no alarm symptoms, carrying endoscopy may have reassuring effect on patient with troubling anxiety over their symptoms. H.Pylori test is causally associated with the majority of peptic ulcers and is the most important risk factor for gastric cancer. Patients with a positive test result should receive eradication therapy, whereas patients with a negative test result should be treated empirically, usually with a PPI. The benefits of this test-and-treat strategy are the cure of PUD or prevention of furure peptic ulcers and symptom resolution in a small subset of patients with functional dyspepsia who are infected with Hp Treatment General measures Reassurance and education are of primary importance in patients with functional dyspepsia. Lifestyle and dietary measures: > Having patients eat smaller, more frequent meals, avoiding meals with a high fat content. > Avoiding coffee! smoking may decrease symptoms. 20 Avoid or substitute aspirin and other NSAIDS. > Refer to psychiatric therapy, If the patient has an apparent coexisting anxiety disorder or depression Patient presenting with dyspeptic symptoms: ‘PP (sandard dove) BID + amexiellin 1000 mg BID for 7 days; then PPI BID + amoxicillin | g BID. + clarithromycin $00 mg BID + metronidazole or Regimen not eudied m Noth America; complexity of regimen makes it ess desirable as firs line tinidazole $00 mg BID for 7 days iherspy Levofloxacin | PPI standard dose BID + eveloxacin 300 mg daly | Lack of North American date; triple + amoxielin 1090 mg BID for 10-14 days ‘once fr high evelonacin resistance rates [Levofloxacin | PPI (Standard or double dox) BID+ amovieilin | Lack of North American data; may be sequrtial | 1000 mg BLD for $7 days; thea PPL BID + more effecive than levofloxacin triple amoxil 1900 mg BID + evorkxacin soomg | merspy daily + metrunidazae otnidszole $00 mg BID for 5-1 days LOAD PF (double dose daly + levooxacin 250 mg daily + metrnidazale otaidazole $00 mg BID + doxycycline 100 mg daly Tack of Nerth American data; nay be more effecve than levofloxacin ple therapy 2 If heart burn the major complain: treat as GERD > Ifon NSAIDs or COX-2 inhibitor : discontinue /switch / add PPi > [fage 60 or older/alarm sign H.Pylori treatment arty EGD 1, General recommendations, based on the American College of Gastroenterology (ACG) guidelines, are to include an antisecretory agent (preferably a PPI) plus at least two antibiotics (clarithromycin and amoxicillin or metronidazole) in the eradication regimen. Regimen Drugs and doves Noles | Chritronycia | PP standard or double dose) BID + moxicitin voi iftarituomycin resistance rate tile 1000 ng BID ormetrenidezo S10 mg TID + is krowntobe> 15% or inpatients | clacton Mm BID fr 1 days withprer marae expose [Bimmeh | PP(sundard dose BID+ Bismuth sibsalclate | ital teatmentchiceit Jquadrmpe {300mg QUD ce Bismsh saberate 120-300 ng | clarithromycin esisance res are (QUD + metoniazsle290QID orS mg TID-QID_ ighor aint fas priormasrlde + ttrayline 00mg QID*-+PPIBID x10 days exposure | Concomitant —|P (standard dose) BID + clarithromycin SO0 mg | Appearsat kastas effective as BID +amovilin 10 mi BID + metronidzale or clarithromycin tpl therapy, tough tinidazae 50 mg BID for 1014 days rol validated in Noth America [Sequential | PP atundard dose BID-+amoiilin 1000 mg BID | Efccysubjstto geographic or 57 das; hen PPI BID + clrthromein 500mg | variion of stance ates, BID + mevonidaoleotinidazole $00 mg BID or | complexity of egien makes it ks 5 days desinble as sti therapy Proton pump inhibitors They bind to the H+/K+-ATPase enzyme system (proton pump) and suppress the secretion of hydrogen ions into the gastric lumen These drugs include: Omeprazole, Lansoprazole,Esomeprazole. B Rabeprazole, _Pantoprazole, Greater degree of acid suppre: n achieved and typically longer duration of action than H2 receptor blockers. PPIs should be taken 30 to 60 minutes before breakfast or the largest meal of the day Adverse reactions Overall, well tolerated; possible adverse effects include headache, dizziness, nausea, diarrhea, and constipation. Increased short term risk of Community-acquired _ pncumonia, Clostridium difficile-associated diarrhea, increased risk of hip fracture, Hypomagnesemia. Long-term PPls use was associated with increase in gastric cancer risk in H. pylori-infected subjects who had received eradication therapy. The risk of gastric cancer increases with the dose and duration of PPIs use Drug interactions Inhibition of CYP450: Omeprazole inhibits the metabolism of substrates, such as diazepam, phenytoin, and warfarin Potential reduced effectiveness of clopidogrel through inhibition of CYP2C19-mediated conversion to active metabolite by omeprazole Recommendations from the U.S. Food and Drug Administration (FDA) are to avoid omeprazole and to use pantoprazole as an alternative. May also interfere with drugs that depend on gastric pH for absorption. (e.g., ketoconazole, itraconazole, protease inhibitors, ampicillin, vit B12). Interaction with high-dose intravenous methotrexate. PPIs inhibit methotrexate renal excretion. Patients receiving high-dose intravenous methotrexate should avoid PPI use, with a switch to ranitidine if needed Holding the PPI dose for 2 days before and after methotrexate administration can prevent the interaction 2 TCAs Individuals with FD demonstrate visceral hypersensitivity and abnormal central pain processing. * v v Mechanism of action is not yet clear, but Antidepressant therapies have been shown in multiple trials to reduce symptoms in irritable bowel syndrome. There is a large overlap between irritable bowel syndrome and FD so it is plausible that antidepressants will also be effective for dyspepsia symptoms. TCAs are associated with adverse events (which include constipation, dry mouth, urinary retention, and somnolence) and a significant proportion of patients might prefer not to take antidepressant medication. It should be noted that TCA is recommended before prokinetic for treatment of FD based on the superior evidence for TCA in this indication. Selective serotonin reuptake inhibitors and selective noradrenaline reuptake inhibitors are reported to be no better than placebo. Amitriptyline, desipramine, Nortriptyline, and Mirtazapine are among the agents used. Prokinetics Metoclopramide/ Domperidone > v v v Mechanism: D2 antagonists increase lower esophageal sphincter tone and stimulate contractions of the stomach and small intestine > The D2 antagonisis prevents nausea through central inhibition of the vomiting center (chemoreceptor trigger zone). The effects of metoclopramide and domperidone are largely confined to the upper GI tract, with minimal effect on the colon. Adverse of Metoclopramide include dizziness, fatigue, drowsiness, extraeffects pyramidal symptoms (EPS), and hyperprolactinemia. Domperidone does not cross the blood-brain barrier as readily as metoclopramide; therefore it lacks the extrapyramidal side effects associated with metoclopramide. 5 > Both of these drugs prolong the QT interval and must be used with caution Mosapride/ Erythromycin Know the Signs of TD Symptoms May Range from Mild to Severe Mosapride: > Mechanism: acts selectively on 5-HT4 receptors in the myenteric plexus to stimulate the release of acetylcholine from cholinergic nerves, thus promoting GI motility. > Has no action on D2 receptor so devoid of EPS and adverse cardiac effects. Erythromycin: Has multiple effects on upper GI motility, increasing lower esophageal pressure and potent contractile agent of the upper GI tract (stomach and duodenum ) through action on motilin receptor ‘Therapy Functional dyspepsia subtypes Epigastric pain syndrome Postorandial distress syndrome Reassurance : 7 Prokinetics . “ Fundie relavore ‘rieycic antidepressants ” Rifaximin Psychological therapy Evidence for psychological therapy in functional dyspepsia is limited. However, for patients with a strong psychological component, offering cognitive behavioral therapy is reasonable. General Measures FOR FD > Lifestyle and dietary measures. > Having patients eat smaller, more frequent meals. Pa > Because the presence of lipids in the duodenum enhances gastric sensitivity, avoiding meals with a high fat content may be advisable. > Similarly, consumption of spicy foods containing capsaicin and other irritants is often discouraged > Cessation of smoking and consumption of alcohol are thought to be helpful. Antacids * Antacids are weak bases that react with gastric acid to form water and a salt to diminish gastric acidity. Because pepsin is inactive at a pH greater than 4, antacids also reduce pepsin activity. * Include Magnesium hydroxide, aluminum hydroxide, Sodium bicarbonate, Calcium carbonate, * They should be administered after meals for maximum effectiveness Antacids adverse effects: * Aluminum hydroxide tends to cause constipation © Magnesium hydroxide tends to produce diarrhea. + Absorption of the cations from antacids (Mg2+, AI3+, Ca2+) is usually not a problem in patients with normal renal function; however, accumulation and adverse effects may occur in patients with renal impairment. Questions 1- What is the optimum treatment for a peptic ulcer caused by H.Pylori A- Proton Pump inhibitor (e.g. pantoprazole) and 2 antibiotics (metronidazole and clarithromycin) B- A proton pump inhibitor C- H2 antagonist D- Antibiotics 2- 80% of people with dyspepsia will have a normal endoscopy - what is this called? 2B A- Functional dyspepsia B- Uninvestigated dyspepsia C- Dyspepsia with alarm symptoms D-Uncomplicated (or simple) dyspepsia 3+ Which of the following is NOT an alarm feature of dyspepsia? A- weight loss B- pale stool C- Mclaena D-Dysphagia E- Vomiting 4- Which of the following is the most important, productive, and practical reason to offer or refer for upper endoscopy in any patient over 60 years with persistent dyspepsia? A. To diagnose gastric peptic B-To diagnose gastroesophageal C-To diagnose early stomach D-To diagnose peptic duodenal ulcer E- To diagnose atrophic gastritis ulcer reflux cancer disease Inflammatory Bowel Disease ¢ IBD ~ chronic intestinal inflammation © Ulcerative Colitis © Crohn’s Disease © Ulcerative colitis - nonspecific inflammatory bowel disease of unknown etiology that effects the mucosa of the colon and rectum © Crohn’s disease - nonspecific inflammatory bowel disease that may affect any segment of the gastrointestinal tract Indeterminate colitis: 15% patients with 13D impossible to differentiate Ulcerative Colitis > Superficial mucosal inflammation of colon only > Begins at rectum and spreads continuously > 30% proctitis, 40% L sided colitis, 30% pancolitis > Symptoms: bloody diarrhea, tenesmus (a feeling of incomplete defecation), abdominal cramping Crohn’s Disease Transmural inflammation of any part of GI tract, presence of “skip” lesions Rectum often spared 30% small bowel (usually terminal ileum), 40% ileum/colon, 25% colon, siomach/duodenum Symptoms: non-bloody diarrhea, weight loss, fever, Right Lower Quadrant pain and/or mass, perianal disease with abscess and/or fistulas 30 UC vs. CD uc cD Continuous/superficial “Skip"/Deep Colon only w/ rectum Mouth to anus+rectum, Rectal bleeding ’Rectal bleeding Rare fistulas/strictures ++fistulas/strictures Surgery curative Surgery palliative (high rate of recurrence, 50%) ern e aL Wu o Ater on ota) See aed rae? infection) 31 Crohn’s Symptoms > Active Crohn's Chronic or noctumal diarrhea Rectal bleeding Abdominal pain Weight loss Fever Fatigue Extraintestinal Manifestations Skin/eyes/joints > Previous disease activity predicts future disease activity PAAR YN Full year of remission followed by 80% chance of remission in following year IBD Treatment Goals * Induce and maintain response/remission Prevent complications Disease related Therapy related Improve/maintain quality of life © Limit surgery * Mucosal healing Principles of Treatment © Treatment of active disease followed by maintenance of remission © One size does not fit all © Risks vs benefit Treatment Overview » Induction 2 Sulfasalazine Mesalamine Steroids Azathioprine/6-MP Methotrexate Biologics > Maintenance Sulfasalazine Mesalamine Azathioprine/6-MP Methotrexate Biologics Medical Therapy > Aminosalicylates (e.g. sulfasalazine, mesalamine) Block cyclooxygenase, lipoxygenase pathways of arachadonic acid metabolism, scavange free-radicals > Corticosteroids (c.g, prednisone, budesonide) Block phospholipase A2, thereby decreasing prostaglandins and leukotrienes PR administration effective in rectal and left-sided disease with fewer adverse systemic effects > Immunomodulatory agents: 6-MP, Azathioprine - inhibit proliferation of T>B lymphoctyes 5-ASA Risks > Sulfasalazine - Headache - Nausea/vomiting ~ Rash - Folate malabsorption - Reversible oligospermia - Pancreatitis - Bone marrow suppression - Interstitial nephritis > Mesalamine - Headache - Nausea = Rash - Pancreatitis = Interstial nephritis Corticosteroids + IV: hydrocortisone, methylprednisolone * Orak: prednisolone, prednisone, budesonide + Budesonide — glucocorticoid — Limited systemic bioavailability + Less toxicity — Ileal and right sided colonic disease — Short term efficacy less than conventional steroids (~15%) — Best combination of short term efficacy and safety Corticosteroids Risks © Cataracts © Glaucoma 34 Diabetes Weight gain Hypertension Osteopenia/Osteoporosis Acne * Mood/sleep disturbances Infection Immunomodulator drugs Azathioprine/6-MP: are purine analogues that interfere with nucleic acid metabolism; reduce cell proliferation and have immune modifier properties Azathioprine Risks + BM — Leukopenia (2-5%) + Hepatotoxicity (rare) + Pancreatitis (3%) + Drug intolerance (10-15%) — Fatigue — Nausea — Flu-like Hypersensitivity reactions + Lymphoma (~4x) Methotrexate + Well documented effectiveness in steroid dependent Crohn's + competitively inhibits dihydrofolate reductase (DHFR), an enzyme that participates in the tetrahydrofolate synthesis. MTX Risks > Nausea 5 Fatigue/malaise Hepatotoxicity Abni LFT’s ~25% Fibrosis/cirrhosis rare BM suppression Hypersensitivity pneumonitis 1% of patients > Teratogen Increased risk of infection Lymphoma risk is rare > > > > > > > > > > Anti-TNF’s 1-Infliximab FDA approved 1998 © Mouse chimeric monoclonal Ab IV (0, 2, 6, then q8wk) 5 mgkg 2-Adalimumab FDA approved 2007 ¢ Fully human monoclonal Ab © SQ q2wks (loading 4 pens, then 2 pens, then © Each pen 40 mg 3- Certolizumab © FDA approved 2008 ae Hong 1 pen) © Pegylated humanized Fab fragment © SQ 0,2, 4, then q4wk) © 400 mg Benefits Steroid free remission Improved quality of life Decreased hospitalizations Decreased need for surgeries Improved mucosal healing vvvVY Risks of Anti-TNF’s + Infusion Reaction (5%) + Infection — Reactivation TB, HBV — Sepsis + Lymphoma + Demyelinating disorders (rare) + Hepatotoxicity (rare) + Drug induced lupus (<1%) 4- Natalizumab: is a fully humanized antibody against alpha-4 integrin Prevents inflammatory white blood cells from migrating into tissue Can be used when patient has failed at least one anti-TNF medication Cannot be combined with other immunosuppressants Must taper off steroids within 6 months Administered intravenously Given every 4 weeks vVVVV Vv » Dose is 300 mg ROC Infliximab LE 4 Prednisone, _ .__ AZAI6-MPI ee eerie) 9 ves ED Cie EYEE ea EL Se Mek eT Early aggressive biologic therapy vs eeu C ne Cue Puke nea Lect ope Ear re remission occuring more rapidly & longer time period to relapse decreased need for treatment with corticosteroids faster reduction in clinical symptoms rapid decline in biochemical inflammatory markers (C-reactive protein) & improved endoscopic mucosal healing 39 Constipation Definitions A symptom-based disorder defined as unsatisfactory defecation and. is characterized by Infrequent passage of stool (less than 3 stools per week) , Hardness of stool, Or Other symptoms (eg, excessive straining, a feeling of incomplete evacuation, a sense of anorectal blockage during defecation, and the need for manual maneuvers during defecation), which suggest a defecatory disorder. Abdominal pain and distension can occur, as well as low back pain and anorexia. Epidemiology Symptoms of constipation are extremely common; the prevalence is approximately 16% in adults overall and 33% in adults older than 60 yea Women Are More Prone To Develop Constipation Risk factors + Gender Age Low levels of income and education Certain medications Certain medical disorders Diet / lifestyle Low level of physical activity 40 Gender ** The prevalence of constipation among females is 2 to 3 times higher than males. + Causes include longer colon transit time in females (especially during luteal phase when estrogen levels are highest) that has been linked to estrogen and progesterone action on smooth muscle motility. Age * The prevalence of constipation among older adults is 15% to 30% higher than younger adults. + Constipation complaints are associated commonly with excessive need for straining and hard stool rather than decreased stool frequency. + Main causes include poor diet , decreased bulk intake , decreased fiber and fluid intake, decreased exercise , and weakening of pelvic and abdominal muscle responsible for voluntary control of defecation. * Others include chronic illness , medication, and psychological factors. Secondary Constipation > Opioid-induced » Endocrine/metabolic conditions (diabetes mellitus, hypercalcemia, hypokalemia, hypomagnesemia, hypothyroidism, pregnancy) > Myopathies > Neurogenic conditions (brain trauma, stroke, Parkinson disease , multiple sclerosis, spinal cord injury or tumor). > Mechanical obstruction (colon cancer) > Medications (analgesics, anticholinergics, antidiarrheals, antihistamines, some antipsychotics and antidepressants.) > Nutritional causes Pathophysiology: Secondary Constipation a1 + Symptoms of constipation may be secondary to diseases of the colon (stricture, cancer, anal fissure, proctitis), metabolic disturbances (hypercaleemia, hypothyroidism, diabetes mellitus), and neurologic disorders (parkinsonism, spinal cord lesions). + Some of these will be amenable to specific therapies, but when they are not, the challenge remains one of symptomatic treatment of constipation. Box 7. Common Causes of Drug-Induced Constipation ‘Aluminum-containing drugs (antacids, sucralfate) Primary Antihistamines Constipation Benzodiazepines > Functional Bileacid sequestrants, constipation, known Calcium channel blockers as chronic Calcium supplements and antacids idiopathic Diuretics constipation (CIC), Tron supplements is constipation that Opioids does not have a Phenothiazines physical Scopolamine, benztropine (anatomical) nor Tricyclic antidepressants metabolic cause but may havea neurological, psychological, or psychosomatic Symptoms of constipation may be secondary to diseases of the colon (stricture, cancer, anal fissure, proctitis), metabolic disturbances (hypercalcemia, hypothyroidism, diabetes mellitus), and neurologic disorders (parkinsonism,spinal cord lesions). > Some of these will be amenable to specific therapies, but when they are not, the challenge remains one of symptomatic treatment of constipation > Symptoms of constipation may be secondary to diseases of the colon (stricture, cancer, anal fissure, proctitis), metabolic disturbances (hypercalcemia, hypothyroidism, diabetes mellitus), and neurologic disorders (parkinsonism, spinal cord lesions). v 22 > Some of these will be amenable to specific therapies, but when they are not, the challenge remains one of symptomatic treatment of constipation. Defecatory Disorders > These disorders are primarily characterized by impaired rectal evacuation from inadequate rectal propulsive forees and/or increased resistance to evacuation > Manual rectal evacuation using a finger, position changes or frequent enema use is common. > Frequently, laxatives are highly ineffective, and DD patients may even have difficulty evacuating liquid stools. “3 Normal-transit constipation > NTC (otherwise known as ‘functional’ constipation) is the most common subtype of primary constipation. Although the stoel passes through the colon at a normal rate, patients find it difficult to evacuate their bowels. In this constipation such as the presence of hard stools or a perceived difficulty with evacuation. However, on testing, stool transit is not delayed, and the stool frequency is often within the normal range v v ituation, patients report symptoms they believe are consistent with v Slow-transit constipation * Slow-transit constipation (STC) causes infrequent bowel movements (typically less than once per week). + Often, patients do not feel the urge to defecate and may complain of associated bloating and abdominal discomfort. As the name suggests, colonic transit time is prolonged in these patients, + A patient must have experienced at least two of the following symptoms each for >1/4 (25%) of defecations except (e) over the preceding 3 months : (a) Straining on passage of stools. (b) Hard stools. (c) Sensation of incomplete evacuation. (d) Feeling of anorectal obstruction. (¢) Fewer than three defecations per week + Loose stools rarely present without use of laxatives. * Does not meet Rome IV Criteria for IBS. “4 Diagnosis Clinical Evaluation Historical features are key, and the questioning of the patient must be specific. What feature does the patient rate as most distressing? Is it infrequency per se, straining, hard stools, unsatisfied defecation, or symptoms unrelated to bowel habits or defecation per se (eg, bloating, pain, malaise)? A full record of prescription and over-the counter medications must be obtained. Evaluation of psychosocial status is recommended because constipation may occur in patients who are depressed or in psychosocial distress. + End oscopic evaluation is required in patients with weight loss, rectal bleeding, or anemia to exclude cancer or strictures, especially in patients older than 50 years. Anorectal examination, manometry, Defecography, colonoscopy, Colon transit measurement and other procedures may be useful in certain circumstances. The presence of these last characteristics suggests underlying irritable bowel syndrome “5 Referral and further investigations 1 A awe YD Symptoms for more than 1-2 weeks despite treatment Considerable pain or cramping Pregnancy Presence of fever Blood in the stool Reduction in stool caliber Weight loss Neurological disorders : Paraplegia, quadriplegia Treatment Non-pharmacological Increase fluid intake to 6-8 glasses of water per day, although minimal evidence to support efficacy if dehydration is not present Increased dietary fiber intake. High-fiber foods include beans, fresh fruits, and vegetables. Incorporate or increase exercise to 3-5 days/week. Pharmacological + Choose drug therapy on the basis of desired onset of action, patient preference, presence of potential contraindications, and use in special populations. + Provide patient education on alternative dose forms (enema, suppository). Bulk forming laxatives They are either naturally derived (psyllium /bran), semisynthetic (polycarbophil), or synthetic (methyleellulose). Definition : Fiber is that part of food that resists enzymatic digestion and reaches the colon largely unchanged. 46 arta asi PROPERTIES OF DIFFERENT DIETARY feta oma ais WATER SOLUBILITY | % FERMENTED Nonpolysaccharides Lignin Cellulose In general, insoluble, poorly fermentable fibers, such as lignin, are most effective in. iecrensivar tise bull ad treat Fibers solubility and fermentation: Soluble fibers + Fermentation of fiber has two important effects: (1) It produces short chain fatty acids. (2) It increases bacterial mass. Colonie bacteria ferment fib to varying degrees, depending on its chemical nature and water solubility. + Short-chain fatty acids may have a prokinetic effect and increased bacterial mass may contribute to increased stool volume. + Fruits and vegetables contain more pectins and hemicelluloses + Psyllium husk is hydrophilic mucilloid that undergoes significant fermentation in the colon, leading to an increase in colonic bacterial mass; the usual dose is 2.5—4 g/d a7 + methylcellulose (~2 g three times a day) and the hydrophilic resin caleium polycarbophil (I-2 yd), are other available preparations of soluble fibers, These poorly fermentable compounds absorb water and increase fecal bulk. Insoluble fibers “* Mechanism : not fermented by colonic bacteria due to poor solubility so can absorb and attract water and increase stool bulk “ Examples : Bran ( Has high lignin content ). Dose : 1-3 g up to three times a day. Use of Bulk forming laxatives + Onset : 12-72 hr. * Role: Less effective in drug-induced constipation and STC. Several formulations; soluble forms can be incorporated into foods, liquids, and recipes; safe in renal and hepatic disease, pregnancy, geriatrics; Precautions + Requires adequate water intake to be effective; + Adverse effects : Can cause gas and bloating although tolerance develop on long term use in most cases. Osmotically Active Agents + Saline Agents + Non digestible Sugars and Alcohols + Polyethylene Glycol—Elecirolyte Solutions 2B Saline agents + Onset: 1-6 br. + Mechanism: Salts of sodium, magnesium, and phosphate pull water into the lumen of intestines resulting in increased enteral pressure. + Adverse reaction: > Magnesium and phosphate may accumulate in patients with renal dysfunction. > Sodium and phosphate derivatives may cause dehydration, hypernatremia, hyperphosphatemia, acidosis, hypocalcemia, and worsening renal function or CHF. + Role : Acute or intermittent constipation, preoperative or preprocedural bowel preparation Non digestible Sugars and Alcohols ‘mote aatives Glyeerin Management of avute or Suppository fast one within i) intermittent constipation used in petit patents Lactilos: Management of acute, interitent, 8 (nay rquie multiple doses, or chreniccorstigation, inchding | associated wit gas and bloating CIC; preferredinchroni ver | for solution disease roporpnde “9 Polyethylene Glycol-Electrolyte Solutions Polyethylene glycol | Acute or chronic constipation; | Onset 1-3 days, safe in renal and hepatic disease eflctie for CIC preoperative! | and pregnancy; overal, wel tated; may be ued | colon preparation longterm ner Stool Softe * Docusate salts are anionic surfactants that lower the surface tension of the stool to allow mixing of aqueous and fatty substances, softening the stool, and permitting easier defecation. + Ivalso stimulate intestinal fluid and electrolyte secretion * Onset of Action. Oral: 12 to 72 hour * Role: Prevention of opioid-induced constipation in combination with Senna or prevention of straining in post-MI, postsurgical, and pregnant patients. Stimulant laxatives Examples: bisacodyl , Senna . Onset: 1-6 hr. Mechanism: Have direct effects on enterocytes, enteric neurons, and GI smooth muscle and probably induce limited low-grade inflammation in the small and large bowel to promote secretion of water and electrolytes and stimulate intestinal motility. Role: Short-term relief of acute or intermittent constipation or as part of preoperative or colonoscopy bowel preparation Adverse effects: abdominal cramping, electrolyte disturbances, melanosis coli, atonic colon. 50 Enemas Enemas are employed either by themselves or as adjuncts to bowel preparation regimens to empty the distal colon or rectum of retained solid material. Mechanism: Bowel distention by any means will produce an evacuation reflex in most people. Precaution: Repeated enemas with hypotonic solutions can cause hyponatremia. Secretory Agents Lubiprostone Mechanism of Action and Pharmacology. Lubiprostone is a prostanoid activator of Cl- channels. Therapeutic Uses and Adverse Effects. A dose of 8 wg twice daily was found to be effective in constipation-predominant IBS, although higher doses (24 ug twice daily) are given for chronic constipation and opioid- induced constipation. Side effects of Lubiprostone include nausea, headache, diarrhea, allergic reactions, and dyspnea. Precaution : need negative pregnancy test before use Linaclotide Mechanism: Agonist of the GC-C, resulting in enhanced chloride and bicarbonate secretion into the intestinal lumen, leading in turn to water seeretion and enhanced motility. Therapeutic Uses and Adverse Effects. This compound is approved in the treatment of constipation-predominant IBS and chronic idiopathic constipation in adults at doses of 290 jig and 145 pg daily, respectively 51 + Adverse effects : diarrhea, abdominal pain, flatulence, and abdominal distension; + Precaution : pregnancy category C Peripheral opioid antagonist Methylnaltrexone + Mechanism: peripherally restricted MOR antagonist. + Role: FDA approved for OIC. * Available in oral and subcutaneous dosage form. Naloxegol + Mechanism of action: Naloxegol is composed of the MOR antagonist naloxone conjugated to a PEG polymer. This limits blood-brain barrier permeability because it is a substrate for the P-glycoprotein efflux transporter, so it behaves as a peripherally restricted MOR antagonist. * Role: Naloxegol is approved for opioid-induced constipation in adults, given orally 12.5 or 25 mg once per day after discontinuing other laxatives. ; + Precaution: contraindicated with use of strong; CYP3A4 inhibitors. Adverse reaction of MOR ANTAGONIST + Abdominal pain, flatulence, and nausea frequently accompany this treatment. * Serious diarrhea sometimes occurs that requires discontinuing therapy. + Patients with known or suspected GI obstru: perforation. (contra-indication) * Opioid withdrawal may be precipitated in patients with a compromised blood-brain barrier. yn are at increased risk of 52 y v v Vv vv v v Notes to guide treatment The conventional effective, safe, and inexpensive modalities (fluid intake: dietary and supplemental fiber; stool softeners; and saline, stimulant, or osmotic laxatives) should be attempted before agents such as secretagogues or PAMORAS are prescribed. Most guidelines recommend starting treatment following inspection of causes and exclusion of secondary causes or cases requiring referral. Start with nonpharmacological therapy....if proved ineffective treat with bulk laxatives or PEG with electrolytes. Other add on therapies includes stool softener (docusate) / Saline agents / lactulose. Stimulant laxative are used as third line and not to be used in chronic setting Failure of the previous agents warrants the use of newer agents Lubiprostone ! linaclotide (constipation-predominant IBS and chronic constipation). OIC treatment with Lubiprostone and PAMORAS should be started only if other options of laxative prove ineffective. Docusate is uscd in Prevention of opioid-induced constipation in combination with Senna or prevention of straining in _post-MI, postsurgical, and pregnant patient Agents used for bowel preparation or for immediate effects include Magnesium salts / Polyethylene glycol / Bisacodyl/ Enema / Senna. In case of laxative / fibers treatment proving ineffective further testing to diagnose or eliminate STC / defecatory disorder should be carried. In case of Defecatory disorder diagnosis of cause whether organic or functional should be carried and treated accordingly . Refractory STC can be treated surgically with total colectomy with ileorectal anastomosis. After exclusion of organic/psychogenic causes . Pelvic floor retraining by biofeedback therapy rather than laxatives is recommended for defecatory disorders 53 Special population: Pregnancy > Bulk laxatives or (PEG with electrolytes) and stool softeners are probably safe during pregnancy because they are poorly absorbed, > Lactulose and magnesium produets can also be used safely. Special population: geriatrics > Saline laxatives containing magnesium, potassium, or phosphates should be used cautiously in persons with reduced kidney function or heart disease. » Diabetes mellitus (some laxatives contain sugars such as dextrose, galactose, and/or sucrose), Special population Patients with the following conditions should use laxatives only under the supervision of a health care provider: (a) Colostomy; (b) Diabetes mellitus (some laxatives contain sugars such as dextrose, galactose, and/or sucrose); (c) Heart disease (some products contain sodium); (d) Kidney disease; and (e) Swallowing difficulty (bulk-formers may produce esophageal obstruction) 54 45-year-old woman comes into the clinic reporting constipation for the past 2 weeks. She recently started calcium carbonate supplementation per her doctor's advice for the prevention of osteoporosis. She would like tostart one of the “natural” laxatives. Which of the fol- lowing isa bulk-forming agent? A. Cascara sagrada B, Sodium biphosphate C. Polycarbophil D. Sennosides E. Docusate sodium 2. A 72-year-old man has started taking lactulose for chronic constipation not relieved by stool softeners. He ‘takes 30 mL tid and over the past couple of days he has had two or three loose bowel movements a day. Which of the following would be the appropriate next step? A. Discontinue the lactulose as it is not indicated for chronic use. B. The lactulose is a hyperosmotic and is too potent. Discontinue the lactulose and start oral senna instead. Switch to mineral oil as it is more mild. Hold the lactulose and continue at a lower dose once the diarrhea resolves. Use the lactulose only on an as-needed basis. eon m A 75-year-old man with a history of hypertension, type 2 diabetes, and chronic low back pain is admitted to the hospital for abdominal pain lasting 2 days. He denies fever, chills, or sick contacts. His last bowel movement was 3-4 days ago. On examination, he is afebrile and has moderate left upper and lower quadrant tenderness. An abdominal radiograph reveals large amounts of stool in the colon with no signs of obstruction, He currently takes lisinopril 20 mg/ day, verapamil 240 mg once daily, acetaminophen S00 mg four times daily, oxycodone sustained release 20 mg twice daily, Patients suffers from moderate CKD. Which therapy would best manage this patient’s constipation? A. Sodium phosphate oral solution. B. Bisacodyl suppository. C. Methylcellulose tablets. D. Methylnaltrexone injection Diarrhea Definition > Diarrhea is a symptom of an underlying disorder, not a disease itself. 56 > It is characterized by: 1. Increased stool frequency (usually greater than three times daily), Increased stool weight, Increased liquidity, A YPN Decreased consistency of stools compared with an individual’s usual patternBristol stool chart Clinical picture » Abdominal cramps, flatulence and weakness or malaise may also occur. > Nausea and vomiting may be associated with diarrhea, as may fever. Pathophysiology About 9 L of fluid normally cross the GIT daily. Of this amount, 2 L represent gastric juice, 1 L is saliva, | L is bile, 2L are pancreatic juice, 1 L is intestinal secretions, es ‘Seperate hard lumps, like nuts and | Tet PP SO Wroipas 2 Lae ingested. or | Tee2 sausage-shaped butlumpy | the 9 L. of fluid presented to the Type3 Like a sausage but with cracks | intestine, only on its surface about 150 to 200 mL remain in Wee’ Like a sausage or snake, smooth the and soft stool after reabsorption. DP & soft blobs with clear-cut edges, Type 5 se Any ee (passed easily) event that increases the Fluffy pleces with raggededges, | Types annushy stool amount of fluid retained in the Watery, no solid pieces. stool = may Type7 Entirely Liquid result in 7 diarrhea. Clinical classification Diarrhea can be classified by + Time course : Acute versus chronic + Stool volume + Stool characteristic: Watery vs fatty vs inflammatory. + Pathophysiology : Osmotic / secretory / exudative /altered motility Jcomplex Acute versus chronic Acute lasts less than 4 weeks. Infectious diarrhea most common cause of acute diarrhea which occurs because of transmission of infectious agents via the fecal—oral route. Most viral and bacterial infection clear within a week and persistence of diarthea after a week suggest protozoal infection. Noninfectious causes include drugs and toxins, laxative abuse, ischemic bowel disease, lactase deficiency, intial presentaion of chronic diarrhea as in IBS ‘IBD. Diarrhea or greater. is considered to be chronic when the symptoms persist for 4 weeks Chronic diarrhea usually results from functional or inflammatory bowel disorders, endocrine disorders, malabsorption syndromes, neoplastic diseases and drugs. it may be intermittent or continual. Infection causes may be considered in immunocompromised patients as Cryptosporidium, cytomegalovirus (CMV), and Mycobacterium avium- intracellular complex and protozoal infection Stool volume 58 » The differentiation of diarrheal cause based on stool characteristics depends on the fact that rectosigmoid colon acts as stool reservoir prior to defecati » If the function of his reservoir is compromised by excess motility or inflammation frequent small volume bowel movements take place. » Ifthe function of this is intact, usually distal colon and small intestine may be the cause of the diarrhea which is characterized by less frequent large volume bowel movements as disruption of the function of distal colon and small intestine manifests as defect in absorptive capacity. The differentiation is important in diagnosis of disease as: IBS and IBD and dependent on patient history and reporting. o Watery vs fatty vs inflammatory This classification is important in differential diagnosis of chronic diarrhea. > Watery diarrhea: can indicate abnormal clectrolyte absorption or sceretion or presence of osmotic component to diarrhea (secretory vs osmotic). > Fatty diarrhea can imply abnormal absorption of fats and other nutrients in the small intestine (pancreatic and ileal functions). > Inflammatory diarrhea: guide the clinician towards the investigation of inflammatory process as IBD / neoplastic disorders .. Osmotic Diarrhea > Osmotic diarrhea results from the intake of un-absorbable, water-soluble solutes in the intestinal Jumen leading to water retention. > Diarrhea occurs Secondary to a hyperosmolar gradient in the intestinal lumen > Common causes: Osmotic laxatives, carbohydrate malabsorption (lactase deficiency), fat malabsorption (pancreatic insufficiency), excess Tons intake as Mg (Antacid abuse) Secretory Diarrhea 59 > Secondary to enhanced secretion by intestinal mucosa; often, large, watery volume with loss of electrolytes > Common causes: Bacterial or viral or bacterial enteritis, gastric hyperseeretion, carcinoid, stimulant’ secretory laxatives, bile acid malabsorption, celiac disease, IBD. Altered motility or motor function > Altered motility can lead to diarrhea that has both secretory and osmotic components. > Rapid transit decrease contact time between luminal contents and absorptive surface and diarrhea result despite intact mucosal absorptive capacity. > Diarrhea in this cases is exacerbated by malabsorption of nutrients which produces un osmotic component to diarrhea Slow transit can lead to diarthea as well as it allows for bacterial overgrowth which in turn disrupts both absorption and electrolyte and fluid balance in the intestinal lumen. Common causes: diabetic neuropathy, hyperthyroidism, IBS. ¥ v Exudative o1 flammatory » Inflammatory (or exudative) diarrhea results from changes to the intestinal mucosa that damage absorption processes leading to increased proteins and other products in the intestinal lumen with fluid retention. » The presence of blood or fecal leukocytes in the stool indicates an inflammatory process. Viruses cause a large proportion of cases (rotavirus & adenovirus). Bacterial causes include E coli, Salmonella, Shigella and Clostridium difficile.parasites and protozoa such as Entamoeba histolytica and Giardia lamblia. » Most of these infectious agents can cause traveler’s diarrhea (It has an abrupt onset but usually subsides within 2 to 3 days) Two commonly seen infections are Campylobacter and Salmonella, which are often associated with contaminated poultry. Contaminated eggs have also been found to be a source of Salmonella vv v v 60 Box 6. Common Causes of Drug-Induced Diarrhea Acarbose or miglitol Antibiotics Antineoplastics Colchicine Digoxin Laxatives Levothyroxine (over-replacement) Metoclopramide NSAIDs Prostaglandins (misoprostol) Orlistat Sorbitol (sugar-free products) reheated. infec! ns > B cereus > Is usually associate d- with cooked rice, especiall y if it has been kept warm or has been > are less common but can be severe with toxins being released into the body, which can cause kidney failure Complex diarrhea Complex diarrhea describes a condition where multiple factors interplay to participate the diarrhea neos05 VS un a Epithelium — Muscio a i~ Permeability =—= Transport ——= Motility—=—= wetabolism Drug-induced diarrhea Diagnosis > Patients with diarrhea should be questioned about the onset of symptoms, recent travel, diet, source of water, and medication use. Other important considerations include duration and severity of the diarrhea and the presence of abdominal pain or vomiting; blood in the stool; stool consistency, appearance, and frequency; and weight loss. Although most cases of diarrhea are self-limited, infants, children, elderly persons, and immunocompromised patients are at risk for increased morbidity When to initiate the investigation wv vv v Stool diagnostic studies may be used if available in cases of dysentery. moderate-to-severe disease, and symptoms lasting >7 days to clarify the etiology of the patient's illness and enable specific directed therapy. > Diagnostic evaluation using stool culture and culture-independent methods if available should be used in situations where the individual patient is at high risk of spreading disease to others, and during known or suspected outbreaks. When dealing with persistent (14-30 d) or chronic Evaluate for disease- and drug-induced causes (laxative, recent antibiotic use), Recent travel history, and temporal relation to food intake. Assess fluid and electrolyte status. Assess CBC, stool culture and evaluate for ova and parasites if infectious cause is suspected. Assess for C. difficile toxin and culture if there has been recent antibiotic use or hospitalization Evaluate stool pH, electrolytes, osmolarity, or fat content, if indicated 8 > Evaluate presence of blood by fecal occult blood test imaging (abdominal CT scan) or endoscopy with biopsy may be indicated, particularly for inflammatory diarrhea or suggestion of neoplasm or celiac disease. Referral Referral to higher level of care or further evaluation may be necessary for some patient: Immunocompromised Infants and children Pregnant women Presence of fever Blood in the stool Weight loss (greater than 5%) Treatment Strategies VVVVVVVVY Removal or treatment of underlying causes, if possible Avoid dairy products because transient lactase deficiency can occur. BRAT” dict for adults (mild foods such as bananas, rice, applesauce, toast) Immunocompromised Infants and children Pregnant women Presence of fever Blood in the stool Weight loss (greater than 5%) ORT indication v > The risk of dehydration from diarrhea is greatest in babies, and rehydration therapy is considered to be the standard treatment for acute diarrhea in babies and young children. Oral rehydration sachets may be used with antidiarrheal in older children and adults. Vomiting is not an indication to stop ORT. 64 Rehydration may still be initiated even if referral to the doctor is advised. Each packet contains: 1) Sodium chloride 2) Trisodium citrate dihydrate 3) Potassium chloride 4) Glucose anhydrous Table 6 Amount of rehydration solution to be offered to patients Age Quantity of solution (per watery stool) Under 1 50m (quarter of a glass) 1-5 100ml (half a glass) 6-12 200ml (one glass) Adult 400 ml (two glasses) ORT Mechanism > Sodium chloride & Potassium Chloride: replace electrolyte deficit adequately and safely. > Glucose: enhance absorption of electrolytes and water from the intestine. > Citrate: added to revert metabolic acidosis. Amount given probiotics > The GI tract contains a vast and complex commensal microflora neces for health. > Alterations in the balance or composition of the microflora are responsible for antibiotic-associated diarrhea and possibly other disease conditions. > Competition with pathogenic organisms, production of antimicrobial substances, enhancement of immune response. Strains utilized: Lactobacillus, Saccharomyces spp. Have a role in IBS and IBD Prevention of antibiotic associated diarrhea: adjunctive therapy for treatment of C. difficile; small intestinal bacterial overgrowth. ry vv Bismuth Subsalicylate Mcchanism: Bismuth stimulates absorption of fluid and electrolytes across the intestinal wall (antisecretory action) and the salicylic acid portion inhibiting synthesis of a prostaglandin responsible for intestinal inflammation and hypermotility. Bismuth subsalicylate has antimicrobial activity and can adsorb bacterial toxins. Role: Mild-moderate diarrhea; prophylaxis against traveler's diarrhea Side effects: stool discoloration; avoid in salicylate allergy, age <12 yr., pregnancy, lactation; Caution with anticoagulants; can bind other drugs; can interfere with some radiographic procedures, severe renal insufficiency. Loperamide » Exerts its action through MOR activity, is an orally active antidiarrheal agent. » May also cause indirect or direct inhibition of mucosal secretion. & Role: Mild to moderate noninvasive diarrhea; adjunctive to other non Opiate therapies also effective against traveler's diarrhea, used alone or in combination with antibiotics. 66 > It is used as adjunct treatment in many forms of chronic diarrheal disease with few adverse effects > Dose: The usual adult dose is 4 mg initially followed by 2 mg after each subsequent loose stool, up to 16 mg per day. » If clinical improvement in acute diarrhea does not occur within 48, loperamide should be discontinued. Side effects : constipation, CNS depression (especially in children) Caution: IBD vv Octreotide > Mechanism: Antisecretory > Role: Treatment of tumor associated diarrhea; HIV-associated diarrhea > Dose: administered either subcutaneously or intravenously as a bolus dose. > Standard initial therapy with octreotide is 50-100 jig, given subcutaneously two or three times a day, with titration to a maximum dose of 500 jig three times daily, based on clinical and biochemical responses > A long-acting preparation of octreotide is available for use in the treatment of diarrhea associated with carcinoid tumors and VIP-secreting tumors. } Side effects: depend on the duration of therapy: Transient nausea, bloating, or pain at sites of injection may occur in the short term, and gallstone formation and hypo- or hyperglycemia may happen in the long term. Telotristat Ethyl > Mechanism: This drug reduces diarrhea associated with carcinoid tumors by inhibiting tryptophan hydroxylase, the rate-limiting enzyme of SHT biosynthesis. SHT secretion stimulates fluid secretion and motility in the GI tract > Role: Telotristat is given in combination with somatostatin analog therapy for the treatment of diarrhea in carcinoid syndrome > Dose: A dose of 250 mg po three times/d may be given to adult patients who are not adequately controlled by somatostatin analog therapy alone @7 > Adverse effects: The main adverse effects are constipation, nausea, headache, increased gamma glutamyl transferase levels, depression, peripheral edema, flatulence, reduced appetite. Crofelemer Mechanism: Acts by blocking Cl secretion and accompanying high volume water loss in diarrhea, normalizing the flow of Cl and water in the GI tract Role: > Antidiartheal indicated for noninfectious diarrhea in patients with HIV/AIDS on antiretroviral therapy. > for treatment of short bowel syndrome Dose: This drug is given orally (125 mg twice daily) to adults Adverse effects: include upper respiratory tract infections, cough, flatulence, nausea, joint and back pain. Teduglutide Mechanism: GLP-2 analog Role: Approved for adult patients with short bowel syndrome who are dependent on parenteral support Dosing: 0.05 mg/kg SC qDay Adverse reactios Colonic neoplasms (colonoscopy recommended every 5 yr): intestinal obstruction; biliary/ pancreatic disease (bilirubin, amylase, lipase. alkaline phosphatase, every 6 mo) Drug interaction overview Based on the pharmacodynamic effect of teduglutide, absorption of concomitantly administered oral medications may be increased 68 Monitor patients receiving concomitant oral drugs requiring titration or with a narrow therapeutic index for adverse reactions due te potential increased absorption of concomitant drug Concomitant drug may require dosage reduction Antibiotics > Antibiotics are generally unnecessary as most food-borne infections resolve spontaneously. » The most important treatment is adequate fluid replacement. > Antibiotics are used for Shigella infe: or Campylobacter ones. > Ciprofloxacin may be used in such circumstances. ns and the more severe Salmonella Empirical treatment Should be initiated in case of : + History of travel + Immunosuppression Protozoal infection + Fever Bloody diarrhea + Diarrhea lasting more than 7 days Metronidazole, vancomycin, probiotics and C.Difficle can be considered with protozoal infection Azithromycin can be considered alternative to ciprofloxacin in children and pregnant. Lactase Mechanism: enzymatic breakdown of lactose pr 69 Role: Lactase deficiency or intolerance. Adverse reactions: Well tolerated Adsorbents Adsorbent drugs such as kaolin, pectin, charcoal, and attapulgite do have an effect on form of stools passed, but the number of stools passed and duration of treatment of diarrhea are not shortened and are not recommended 70 Oral H. emic Agents Overview The pancreas is both an endocrine gland that produces the peptide hormones insulin, glucagon, and somatostatin and an exocrine gland that produces digestive enzymes. The peptide hormones are secreted from cells located in the islets of Langerhans Bells produce insulin, acells produce glucagon, and 8 cells produce somatostatin. These hormones play an important role in regulating the metabolic activities of the body, particularly the homeostasis of blood glucose. Hyperinsulinemia (due, for example, to an insulinoma) can cause severe hypoglycemia More commonly, a relative or absolute lack of insulin, such as in diabetes mellitus can cause serious hyperglycemia,which, if left untreated, can result in retinopathy, nephropathy, neuropathy, and cardiovascular complications. Administration of insulin preparations or oral hypoglycemic agents can prevent morbidity and reduce mortality associated with diabetes. Anti-diabetic medications treat diabetes mellitus by lowering glucose levels in the blood. With the exceptions of insulin, exenatide, and pramlintide, all are administered orally and are thus also called oral hypoglycemic agents or oral antihyperglycemic agents. There are different classes of anti-diabetic drugs, and their selection depends on: 1. Nature of the diabetes 2. Age and situation of the person 3. Other factors. Types of DM Diabetes mellitus type 1 is a disease caused by the lack of insulin. Insulin must be used in Type I, which must be injected. 7 Diabetes mellitus type 2 is a disease of insulin resistance by cells. Treatments include: agents that increase the amount of insulin secreted by the pancreas agents that increase the sensitivity of target organs to insulin agents that decrease the rate at which glucose is absorbed from the gastrointestinal tract. Frequency 10-20% 80-90% Age at onset Early (below —_Late (after 40) 35) ‘Type ofonset Abrupt and Gradual & insidious severe Weight Normal Obese HLA Linked to - HLA DR3,4,DQ Family <20% About 60% history Genetic locus ~ Chromosome 6 Pathogenesis Autoimmune Insulin resistance destruction of beta pancreatic cells Tslet cell - antibodies present Decreased Normal or increased insulin insulin Clinical Insulin & ——Dict,exercise,oral management diet drugs.insulin Acute Keloacidosis _ Hyperosmolar coma complications ‘Types of DM Hyperosmolar coma High blood sugars cause severe dehydration, increases in osmolarity (relative concentration of solute) and a high risk of complications, coma and death, Symptoms of Hypoglycemia Lipodystrophy Hypersensitivity B Drug groups Pret Peptidase-IV ery Rete at Cedar Cery Pcie? eM est ay OTe CCG Weis Adverse effects of OH As 1) Insulin secretagogues Useful in the treatment of patients who have Type 2 diabetes but who cannot be managed by diet alone. Best response to OHA is seen in one who develops diabetes afier age 40 and has had diabetes less than 5 years. Patients with long-standing disease may require a combination of hypoglycemic drugs with or without insulin to control their hyperglycemia. A. Sulfonylureas These agents are classified as insulin secretagogues, because they promote insulin release from the B cells of the pancreas. a The primary drugs used today are tolbutamide and the second-generation derivatives, glyburide, glipizide, and glimepiride. Mechanism of action: . Stimulation of insulin release from the B cells of the panereas by blocking the ATP-dependent K” channels, resulting in depolarization and Ca” influx 2. Reduction in hepatic glucose production 3. Increase in peripheral insulin sensitivity. 1. f Insulin release Sulfonytiress ook . | Glucagon release ol a 3. t Insulin receptor sensitivity kt closes —+ membrane depolarization Glucose 1aTP a calcium channels Insulin Caee Granules Calcium influx releases insulin from granules Insulin Pharmacokinetics: * Given orally, these drugs bind to serum proteins * Metabolized by the liver © Exereted by the liver or kidney * Tolbutamide has the shortest duration of action (6-12 hours), whereas the second-generation agents last about 24 hours Adverse Effects: 1. Weight gain 2. Hyperinsulinemia 3. Hypoglycemia These drugs should be used with caution in patients with hepatic or renal insufficiency, because delayed excretion of the drug-resulting in its accumulation-may cause hypoglycemia. Renal impairment is a particular problem in the case of those agents that are metabolized to active compounds, such as glyburide. Glyburide has minimal transfer across the placenta and may be a reasonably safe alternative to insulin therapy for diabetes in pregnancy. B. Meglitinide analogs This class of agents includes repaglinide and nateglinide. Although they are not sulfonylureas, they have common actions. Mechanism of action: 1. Their action is dependent on functioning pancreatic cells. 2. They bind to a distinct site on the sulfonylurea receptor of ATP-sensitive potassium channels, thereby initiating a series of reactions culminating in the release of insulin, 3. However, in contrast to the sulfonylureas, the meglitinides have a rapid onset and a short duration of action. 4. They are are categorized as postprandial glucose regulators. Meglitinides should not be used in combination with sulfonylureas due to overlapping mechanisms of action Pharmacokinetics: * These drugs are well absorbed orally after being taken | to 30 minutes before meals. * Both meglitinides are metabolized to inactive products by CYP3A4¢ in the liver. © Excreted through the bile. 76 Adverse Effects: 1. Incidence of hypoglycemia is lower than that of the sulfonylureas 2. Repaglinide has been reported to cause severe hypoglycemia in patients who are also taking the lipid-lowering drug gemfibro: 3. Weight gain is less of a problem with the meglitinides than with the sulfonyluteas. 4. Must be used with caution in patients with hepatic impairment. 2) Insulin sensitizers Two classes of oral agents-the biguanides and thiazolidinediones improve insulin action. These agents lower blood sugar by improving target-cell response to insulin without increasing pancreatic insulin secretion. They address the core problem in Type II diabetes—insulin resistance. A. Biguanides 1. Metformin (glucophage), the only currently available biguanide 2. it increases glucose uptake and utilization by target tissues, thereby decreasing insulin resistance. 3. Requires insulin for its action, but it does not promote insulin secretion 4. Hyperinsulinemia is not a problem. Thus, the risk of hypoglycemia is far less than that with sulfonylureas Mechanism of action: 1. Reduction of hepatic glucose output, largely by inhibiting hepatic gluconeogenes Slowing intestinal absorption of sugars Improves peripheral glucose uptake and utilization. Metformin may be used alone or in combination with one of the other agents, as well as with insulin. 5. Hypoglycemia has occurred when metformin was taken in combination. RYN Pharmacokinetics: * Metformin is well absorbed orally, is not bound to serum proteins 7 © Itis not metabolized * Excretion is via the urine. Adverse effects: T. These are largely gastrointestinal. 2. Contraindicated in diabetics with renal and/or hepatic disease, acute myocardial infarction, severe infection, or diabetic ketoacidosis 3. It should be used with caution in patients greater than 80 years of age or in those with a history of congestive heart failure or alcohol abuse. 4. Long-term use may interfere with vitamin B,2 absorption. Another group of agents that are insulin sensitizers are the thiazolidinediones (TZDs) ot, more familiarly the glitazones. These drugs do not promote its release from the pancreatic § cells; thus, hyperinsulinemia does not result. ‘Troglitazone was the first of these to be approved for the treatment of Type 2 diabetic, but was withdrawn after a number of deaths due to hepatotoxicity were reported. Presently, two members of this class are available, rosiglitazone. Mechanism of action: 1. Exact mechanism by which the TZDs lower insulin resistance remains to be elucidated They are known to target the peroxisome proliferator-activated receptor-y (PPARy)- c nuclear hormone receptor. 3. Ligands for PPARy regulate adipocyte production and secretion of fatty acids as well as glucose metabolism, resulting in increased insulin sensitivity in adipose tissue, liver, and skeletal muscle. 4. PPARy-RXR complex bind to DNA, promoting transcription of several genes with products that are important in insulin signalling. v Pharmacokinetics 2B Both pioglitazone and rosiglitazone are absorbed very well after oral administration and are extensively bound to serum albumin. Both undergo extensive metabolism by different cytochrome P450 isozymes. Pioglitazone: Renal climination is negligible, with the majority of the active drug and metabolites excreted in the bile and eliminated in the feces. Rosiglitazone: The metabolites are primarily excreted in the urine. Adverse Effects: 1. few cases of liver toxicity have been reported with rosiglitazone or pioglitazone. 2. Weight increase can occur, possibly through the ability of TZDs to increase subcutaneous fat or due to fluid retention. 3. Glitazones have been associated with osteopenia and increased fracture risk. 4, Other adverse effects include headache and anemia. 3) a-glucosidase itors. Alpha-glucosidase inhibitors are oral anti-diabetic drugs used for diabetes mellitus type 2 that work by preventing the digestion of carbohydrates (such as starch and table sugar), Carbohydrates are normally converted into simple sugars (monosaccharides), which can be absorbed through the intestine. Hence, alpha-glucosidase inhibitors reduce the impact of carbohydrates on blood sugar. Acarbose and miglitol are orally active drugs used for the treatment of patients with Type 2 diabetes. m of action: 1. These drugs are taken at the beginning of meals. 2. They act by delaying the digestion of carbohydrates, thereby resulting in lower postprandial glucose levels. 3. Both drugs exert their effects by reversibly inhibiting membrane-bound a-glucosidase in the intestinal brush border. 79 4, This enzyme is responsible for the hydrolysis of oligosaccharides to glucose and other sugars. 5. Consequently, the postprandial rise of blood glucose is blunted. Unlike the other oral hypoglycemic agents, these drugs do not stimulate insulin release, nor do they increase insulin action in target tissues. Thus, as monotherapy, they do not cause hypoglycemia. However, when used in combination with the sulfonylureas or with insulin, hypoglycemia may develop. Pharmacokinetics: 1, Acarbose is poorly absorbed. 2. It is metabolized primarily by intestinal bacteria, and some of the metabolites are absorbed and excreted into the urine. 3. On the other hand, miglitol is very well absorbed but has no systemic effects. 4, It is exereted unchanged by the kidney. Adverse effects: 1. The major side effects are flatulence, diarrhea, and abdominal cramping. 2. Patients with inflammatory bowel disease, colonic ulceration, or intestinal obstruction should not use these drugs. 4) Dipeptidy! peptidase-4 inhibitor 1. DPP-4 inhibitors or gliptins, are a class of oral hypoglycemics that block DPP-4. 2. They can be used to treat diabetes mellitus type 2. 3. The first agent of the class - sitagliptin - was approved by the FDA in 2006. 4. Glucagon increases blood glucose levels, and DPP-4 inhibitors reduce glucagon and blood glucose levels. 5. The mechanism of DPP-4 inhibitors is to inerease incretin levels (GLP-1 and GIP), which inhibit glucagon release, which in turn increases insulin secretion, decreases gastric emptying, and decreases blood glucose levels. Sitagliptin is an orally active dipeptidyl peptidase-IV (DPP-IV) inhibitor used for the treatment of patients with Type 2 diabetes Mechanism of action: Sitagliptin inhibits the enzyme DPP-IV, which is responsible for the inactivation of incretin hormones, such as glucagon-like peptide-1 (GLP-1). Prolonging the activity of incretin hormones results in increased insulin release in response to meals and a reduction in inappropriate secretion of glucagon. Sitagliptin may be used as monotherapy or in combination with a sulfonylurea, metformin or a glitazone. Pharmacokinetics: © Sitagliptin is well absorbed after oral administration. * Food does not affect the extent of absorption. © The majority of sitagliptin is excreted unchanged in the urine. © Dosage adjustments are recommended for patients with renal dysfunction. Adverse Effects: In general, sitagliptin is well tolerated, with the most common adverse effects being nasopharyngitis and headache. pramlintide is an analogue of amylin, a small peptide hormone that is released into the bloodstream by the B-cells of the pancreas along with insulin, after a meal Like insulin, amylin is completely absent in individuals with Type I diabetes. By augmenting endogenous amylin, pramlintide aids in the cellular absorption and regulation of-_—_blood glucose by _ slowing gastric emptying, promoting satiety via hypothalamic receptors, and inhibiting inappropriate secretion of glucagon Amylin reduces the production of glucose by the liver by inhibiting the action of glucagon. a hormone produced by the pancreas that stimulates the production of glucose by the liver. Amylin also reduces appetite. 81 In studies, pramlintide-treated patients achieved lower blood glucose levels and experienced weight loss. Exenatide glucagon-like peptide-1 agonist (GLP-1 agonist) medication, belonging to the group of ineretin mimetics that is approved in April 2005 for the treatment of diabetes mellitus type 2. It is administered as a subcutaneous injection (under the skin) of the abdomen, thigh, or arm, any time within the 60 minutes before the first and last meal of the day Mechanism of Action:- * increases insulin secretion in response to eating meals © suppresses pancreatic release of glucagon in response to eating + slow down gastric emptying, reduce appetite, promote satiety via hypothalamic receptors (different receptors than for amylin). Most people using exenatide slowly lose weight © Exenatide Adverse effects: The main side effects of exenatide use are gastrointestinal in nature, including acid or sour stomach, belching, diarrhea, heartburn, indigestion, nausea, and vomiting 2 Diabetes Cases Question 1 A 24-year-old, white female has recently being diagnosed with insulin-dependent diabetes. The disease is being managed on a split dose of 60/40 insulin suspension, which she injects herself at 8:00 a.m. and 5:00 p.m. She was told to call in if she experiences any strange symptoms, which she does this afternoon, At 2:45 p.m. she is not feeling well and notices that her skin is cool and damp. Her hands are shaking and she is very anxious. What do you tell her to do right away, before having somebody take her to your office? A. Inject 4 TU of her insulin B. Drink a can of diet soda C. Drink six ounces of fruit juice D. Eat a large candy bar E.Eat a cube of sugar The above described symptoms are classical for hypoglycemia. An inexperienced patient needs to be monitored by medical personnel until the blood glucose level is stabilized again. The first priority though is to prevent the blood sugar to drop any further and send the patient into a coma. A six-ounce drink of fruit juice should be sufficient. Since the symptoms suggest hypoglycemia, injecting insulin would worsen the situation. If in doubt if the patient is hypo- or hyperglycemic and there is no possibility for a test, always give sugar first and sce if the patient improves. An increase of a blood glucose level of, for example, 350 is not going to hurt the patient, but lowering a level of 45 is going to send the diabetic into a coma. Since diet soda contains only sugar substitutes it is not going to influence the blood glucose level. A large candy bar could be eaten too, but it may raise the glucose B level higher than required, therefore the fruit juice is a better choice. One cube of sugar is an insufficient amount to raise blood sugar; it will take several cubes. Question 2 A 61-year-old white male presents to your office for the first time. In reviewing his medical records, you note a history of diabetes type 2 onset 4 years ago. He is currently well controlled to normal blood sugars and hemoglobin A1Cs. You do note that his previous blood pressure readings were above normal on multiple visits. In addition to his usual diabetic care, you recommend initiation of drug therapy for his—hypertension. Your drug of choice is A. Calcium channel blocker B. ACE inhibitor C. Alpha blocker D. Thiazide diuretic E. beta blocker ACE inhibitors have been shown to slow the progress of proteinuria in patients with diabetes. Abo they show an increase in renal blood flow saving diabetic kidney from renal insufficiencies Control of their blood pressure is just as important. Your drug of choice should be an ACE inhibitor for any diabetic with hypertension as long as there are no contraindications to giving them this medicine. Question 3 ‘A 62-year-old male with a history of diabetes for twenty-five years and hypertension for ten years is referred to your office by a retired physician. He 84 complains of worsening edema on the right lower extremity during the last two wecks. Vital signs show T 97.8 Pulse 78 RR 20 BP 165/93. Physical examination disclosed periorbital edema and left lower extremity edema 1+ below the knee and right lower extremity edema 3+ below the knee that is nontender to palpation. He denies any fever, chills, or malaise. He is currently on glipizide 20 mg twice a day and amlodipine 10-mg once daily. Laboratory result from two weeks ago shows Na 145, K 4.5, Cl 102, HCO3 25, BUN 14, Creat 1.2, CA 7.2. Glucose 287, HgAIC 8.6, and Albumin 2.5. The U/A test shows 3+ proteins with no WBC or RBC. Question: What is the most appropriate next step in management of his diabetes? AStart the patient on glyburide BStart the patient on insulin therapy C. There is no indication to change his diabetes therapy. His HgA1C shows that his diabetes is under control DStart the patient on metformin E. Repeat the patient's HgbAl¢, result is too old to reflect the current HgbAIC level This patient's diabetes mellitus is poorly managed. Glycosylated hemoglobin (HbA\c) is the best indicator of glycemic control over the previous 3 months. Maintaining an HgAlc of 7 or less lowers the rate of complications of diabetes mellitus. Adding a second class of oral hypoglycemic agent would be the next best choice. Metformin is the drug from the biguanides class; therefore, it is the drug of choice. Starting the patient on glyburide is incorrect because both glyburide and glipizide belong to the sulfonylurea class. Starting the patient on insulin therapy is incorrect because oral therapy should be maximized before starting on insulin. HgAlc of 8.6 indicates poorly controlled diabetes. There is a need to change his diabetes management. There is no need to repeat HgAlc. Glycosylated hemoglobin (HDA 1c) is the best indicator of glycemic control over the previous 3 months. The result is not too old. Question 4 A 38-year-old obese female with type 2 diabetes mellitus presents to your office for follow up visit. Her blood sugar has been well controlled on metformin. She wants to have a baby and wants to know if there is any special thing to do so that her diabetes will not affect her pregnancy. Which of the following is appropriate for her? AL Switch her to insulin B. Switch to glyburide C. Tell her it is not advisable for her to be pregnant because her child will be diabetic D. Increase the dose of her metformin to prevent worsening of her blood sugar during pregnancy E. Continue her present treatment and dose Diabetic patients that are pregnant or attempting to get pregnant, irrespective of whether they are insulin requiring or not, should be switched to insulin therapy for their glucose control. Oral glucose lowering agents are contraindicated in pregnancy. Although a poor glucose control predisposes to increased maternal and fetal risks, it is not a contraindication to pregnancy. A good glucose control is however necessary prior to and during pregnancy. 86 Case A 24-year old white male present to the flight physician assistant’s office for a routine physical. He has been taking flying lessons and is applying for a pilot’s license. Regulations state that he must have a physical examination. Examination discloses a thin male, 74 inches, 140 pounds. Generally, the exam is unremarkable. Routine screening includes a urinalysis, which demonstrates 4+ glycouria. Does this man have Diabetes Mellitus? There is insufficient information to establish a diagnosis. There are other reasons to spill glucose in the urine. You must ask additional questions. ‘You find out that even though he denies any history of diabetes, he admits to: Having incessant thirst Increase hunger Increase urination He has lost 15 pounds over the last three months A glucometer reading revealed a glucose of 490 mg/dl Blood glucose revealed a glucose level of 496 mg/dl. With normal electrolytes and anion gap Does this patient have diabetes? Three ways to diagnose diabetes mellitus: Fasting glucose of > 126 mg/dl Oral glucose tolerance test with a2 hr value of > 200 and one other value in- between or > 200 Grossly elevated serum glucose with classic signs and shock. He does not feel ill and has multiple acti ir patient is in ities planned What should you do for hii New onset Type I diabetics should be admitted because: He needs to start on insulin He requires a lot of patient education 7 Case 32-year old woman who has type | diabetes calls you over the weekend. She has been having some nausea and vomiting for the last 24 hours and can not keep solids down. She has been able to keep liquids down. Her last glucometer reading was 261 mg/dl. She is on a split dose of NPH and regular insulin twice daily. She did not take her morning insulin because a physician had told her never to take her insulin when she is not eating. She ask you for instructions? Many patients do not take their insulin if they are not eating: this is wrong. It is important for IDDM patients and their providers to learn how to manage “sick days Note that one of the most common reasons for patient to go into DKA is poor management of their diabetes when they are sick. Insulin should be taken regardless of whether or not they are eating Adequate carbohyrates should be taken (15 — 20g q2h) The patient can consume regular coke, Sprite, juices @ 4 —6 oz q2hrs They can also have gelatin, crackers, soup Patients that are sick require more insulin NOT less The intermediate or long acting insulin should be given Regular insulin should be given q4h as per a ‘sliding scale” ry Case TT is a 55_year_old female , suffering from Diabetic neuropathy, Hypertension and Hyperlipidemia There is no any past surgical history Questions: Identify the problems and try to solve?? How many risk factors does TT have for coronary disease?? Which one is more risky?? What impact may oral_estrogen replacement therapy expected to exert in lipid profile picture? Diabetic neuropathy: TCAs or SSRIs Hypertension: ACE inhibitors Hyperlipidemia: Statins 88 All 3 are common risk factors for coronary heart discases, Diabetes is the most dangerous Oral estrogen therapy may worsen lipid profile through increasing of serum TGs Mrs. Miller is a 25-year Aftican American woman who was diagnosed to have Type 2 diabetes at her last office visit two months ago. At that time, her blood pressure was 144/86. She saw a dietician two weeks ago and began a low sodium ADA diet. Blood pressure at that time was reported to be 140/86. Today her blood pressure is 134/84. Which of the following is an appropriate treatment plan? a) This is an acceptable blood pressure; however her blood pressure should be closely monitored. The patient should now be started on a thiazide diuretic to achieve a target blood pressure of 130/80 The patient should be started on an ACEI to achieve a target blood pressure of 130/80 It is reasonable to give this patient a trial of life style modification prior to initiating pharmacologic therapy. b) ¢ q) D Appropriate treatment for blood pressures of 130-139/80-89 includes lifestyle modification such as weight reduction, reduction in dietary sodium, adopting the DASH ating plan, increasing physical activity, and moderating alcohol consumption. These interventions can decrease systolic blood pressure from 2-20 mmHg (JNC VII). Patients with blood pressures that are consistently above 130/80 mm Hg despite lifestyle modification should also be started on pharmacologic treatment. 40 year old woman with diet controlled Type 2 diabetes is seen for evaluation. After completing your history and physical examination and reviewing previous records you feel confident the patient has no evidence of end-organ damage. Laboratory studies reveal a Hgb Alc of 6.5% and LDL cholesterol to be 120. Which of the following would you recommend? a) Begin clopidogrel to prevent cardiovascular events. b) Begin 325 milligrams ASA daily to decrease cardiovascular events. c) Begin aspirin and lovastatin d) Begin lovastatin and clopidogrel to prevent cardiovascular events. c Aspirin (75-325 mg/d) is recommended in all adult patients with diabetes and macrovascular disease. It should be considered in patients older than forty with diabetes and possibly as young as thirty with additional cardiovascular risk factors Aspirin is contraindicated in patients less than 21 years of age secondary to concerns about Reyes Syndrome. Clopidogrel! should be considered in patients who are aspirin intolerant. 30 Hyperthyroidism and hypothyroidism Stress Cold Cellular effects of thyroid 1No-)K—ATRose 1 Mitochondria + Other enzymes eer sem ery > eg TUS 1 Metabolic 1 Thermogeness 1 Cerdiac ouput Sremnegs Noni i [Tiiete ody offecte ] Thyroid disorders Hyperthyroidism occurs in all ages, uncommon under the age of 15 and 10 times more common in women (1/10,000). Graves disease is the most common etiology, caused by autoimmune thyroid- stimulating antibodies and associated with diffuse goiter, opthalmopathy, and local dermopathy Ophthalmopathy 1, Present in 50% of patients Eyelid retraction . Periorbital edema Proptosis (exopthalmos) Diplopia WRN Hyperthyroidism resembles a state of increased adrenergic activity despite a normal or low serum cortisol level Classic complaints include heat intolerance, palpitations, weight loss, sweating, nervousness, and fatigue 1-Graves’ Disease Autoimmune disease caused by thyroid stimulating immunoglobulins bind to TSH receptors on thyroid Causing hypersecrection of thyroid hormone , hypertrophy & hyperplasia of thyroid follicles. Group Anti-TSHR Ab Anti-Tg Ab General population o 5-20 Graves’ disease 80-95 50-70 Autoimmune thyroiditis 10-20 80-90 Relatives of patients with o 30-50 autoimmune thyroiditis Type | diabetes a 30-40 Pregnant women 0 about 14 Anti-TSHR Ab = Antithyrotropin receptor antibodies Anti-Tq Ab = Antithyroglobuln antibodies Anti-TPO Ab = Antithyroid peroxidase antibodies. Anti-TPO Ab 8-27 50-80 90-100 30-50 30-40 about 14 Pathogenesisis of grave's disease: Excess production of thyroid hormone is caused by the activation of thyrotropin receptors by thyroid-stimulating antibodies produced within and outside the thyroid gland. The intrathyroidal inflammatory cells also produce cytokines, such as interleukin- 1, and interferon-{gamma}, that induce the expression of adhesion molecules such as CD54, regulatory molecules such as CD40, which in turn activate local inflammatory cells. These cytokines also induce thyroid cells to synthesize cytokines that may help sustain the intrathyroidal autoimmune process. Antithyroid drugs reduce the production of thyroidal cytokines -- an ability that may explain their immunomodulatory effects (which include a decrease in the production of thyroid-stimulating antibody) -- contributing to remission in some patients. Clinical Manifestations of Graves’ Disease Panel A shows diffuse goiter in a 28-year-old woman with Graves! hyperthyroidism. Panels B and C show ophthalmopathy in a 55-year-old woman with Graves’ disease, with periorbital edema, and proptosis Panel D shows localized dermopathy Graves treatment: Antithyroid drugs (Thionamides) a, Propylthiouracil (PTU) 300-400 mg daily b. Methimazole 30-40 mg daily ©. Decrease synthesis of hormone, PTU also decreases conversion of T4 to T3 d. Risk of agranulocytosis ¢. PTU used in pregnancy Beta-Blockers for symptoms Thyroidectomy Rapid cure but requires thyroid replacement Radioactive Iodine Iodine (1311) is given Effect is typically seen in 3-6 months Hypothyroidism often develops 2- Multionodular goiter: Less common than Graves and affects older individuals * Discrete nodules become autonomous and hyperfunction © Treatment with thyroidectomy and thionamides Case study A 50-year-old man presents with enlargement of left anterior neck. He has noted increased appetite over past month with no weight gain, and more frequent bowel movements over the same period. Physical exam: He is 5'8" tall and weighs 150 Ib. The heart rate is 82 and the blood pressure is 110/76. There is an ocular stare with a slight lid lag. The thyroid gland is asymmetric to palpation, weighing an estimated 40g (normal = 15-208). There is a 3x2.$ em firm nodule in left lobe of the thyroid What do you think the patient's primary problem is? Probable hyperthyroidism The history of increased appetite (without weight gain) and increased bowel motility is classic for hyperthyroidism. The resting heart rate is mildly elevated, which is consistent but is a common finding in physician's offices. The findings of an ocular stare, lid lag, and an enlarged thryoid are also consistent with hyperthyroidism. The orbital symptoms noted here are most typically associated with Grave's disease and result from inflammation and swelling of retro-orbital tissues However, in this case the thyroid is asymmetrical and contains a nodule, whereas the thyroid gland in Grave's disease is symmetrically enlarged and homogeneous. Initial evaluation of a patient's thyroid status is most commonly done by measuring thryoid stimulating hormone (TSH) concentration in the serum, which is a sensitive indicator of the body's perception of its own thryoid status. In hyperthyroidism, TSH concentration is markedly decreased (the converse is true in hypothyroidism). 96 Usually, serum free T4 will be measured as well and can serve as a confirmatory test for the TSH findings. In some labs, fice T4 assay is not routinely available: similar data can be derived from measuring total T4 and T3 resin uptake, and calculating the Free Thyroxine Index (FTI). ~—s which ~—is__—proportional«=sto fee. = T4. In this patient, there is a nodule associated with the thyroid gland. This finding might result from thyroid pathology but might also indicate a parathyroid adenoma (which would appear in the same area). Thus it is prudent to check the patient's calcium status Active parathyroid adenomas produce hypercalcemia (due to active bone resorption) and may also be associated with marked elevations in circulating alkaline phosphatase. Case study The test results for this patient were as follows (S = measured in serum) Patient's value Reference range Calcium, total (8) 10.6 mg/dl 84 - 10.2 Phosphorus 48 mg/dl 27 - 45 Alkaline phosphatase (S)—s160,s U/L “aw 4 - 120 74, Total (S) 12.2 ugidl 5 : 115 T3 resin uptake (s) 35% 25 35 TA, Total () 31 ng/dl 100 = 215 TSH (s) <0.1 uuiml 07 -1.0 Free thyroxine index (FTI 146 6 = 115 How would you interpret these results? The most important result is the strongly suppressed TSH. The remainder of the thyroid tests is also consistent with hyperthyroidism (elevated FTI and T3). The tests for parathyroid problems rule out a parathyroid process (though the alkaline phosphatase is_~—sonly~—svery_—milldily-—_ elevated) What additional tests would you order? Additional testing should directly address The possibility of Grave's disease and should also determine the nature of the nodule associated with the thyroid. Grave's disease is strongly associated with the presence of anti-thyroid microsomal antibodies, while other antibodies against thyroid epitopes (e.g.. thyroglobulin) occur in Hashimoto's thyroiditis. Furthermore, the thyroid hyperfunction that occurs in Grave's disease can be assessed directly by measuring the rate radio-iodine uptake into the thyroid gland. Case study Serum was obiained for anti-thyroid antibody testing and the following results were obtained. Patient normal antithyroglobulin Ab. neg. neg. antimicrosomal Ab. pos. neg. A thyroid scan to evaluate the uptake of radioactive iodine into the thyroid gland showed 68% uptake at 6 hr. (normal 5 - 28% uptake at these time points The radio-iodine uptake was homogeneously increased over the entire gland except in the area of the palpable nodule, where uptake was decreased. How would you interpret these additional tests? Consistent with grave's discase The anti-thyroid antibody tests and radio-iodine uptake results make a diagnosis of Grave's disease solid at this point. However, the finding that radio-iodine uptake is decreased in the area of the nodule suggests that there is an additional problem in the thyroid gland that from Grave's disease. What would you wish to do next to fin: ize the diagnosis? The finding of a low radio-iodine uptake into the palpable nodule suggests that a thyroid neoplasm might be present. 98 A tissue diagnosis is needed to fully evaluate that possibility, so a fine needle aspirate (FNA) of the nodule was made and the cytology of the recovered cells was examined. The diagnosis from the FNA was papillary carcinoma of the thyroid, and the final diagnosis for the patient wa Grave's disease with papillary carcinoma The patient underwent surgical thyroidectomy followed by thyroid hormone replacement therapy. Later, he was scanned for residual thyroid tissue, which was ablated with iodine- 131. He underwent periodic serum thyroglobulin analysis and iodine-131 scans, which remained negative over a two-year course. Recently, ocular tearing and itching with proptosis were noted on physical exam. Are these ocular_symptoms suggestive of _a_recurrent_thyroid problem .g..recurrent tumor in this patient?) No, remember that Grave's disease is a systemic autoimmune process that has hyperthyroidism as one of its manifestations. The removal of the thyroid gland cures the hyperthyroidism, but not the other symptoms of Grave's disease--which include the ocular symptoms. 99 Corticosteroids History Synthesis Pharmacological Actions Pharmacokinetics Preparations Therapeutic principles Dosage schedule & Steroid withdrawal Uses: Therapeutic Diagnostic Adverse reactions Contraindications Precautions during therapy Glucocorticoid antagonists Pharmacokinetics Absorption: all are rapidly & completely absorbed (Except DOCA) Transport: Transcortin 75% Albumin 5% Free form 20% Metabolism: * by liver enzymes, conjugation & excretion by urine partly excreted as 17-ketosteroids * ty: 0f cortisol 1.5 hours Preparation: * Glucocorticoids = Short acting = Intermediate acting 100

You might also like